LSAT and Law School Admissions Forum

Get expert LSAT preparation and law school admissions advice from PowerScore Test Preparation.

User avatar
 Dave Killoran
PowerScore Staff
  • PowerScore Staff
  • Posts: 5853
  • Joined: Mar 25, 2011
|
#41346
Complete Question Explanation
(The complete setup for this game can be found here: lsat/viewtopic.php?t=10632)

The correct answer choice is (D)

Answer choices (A), (B), and (E) would force two students to be assigned to both Tuesday and Thursday, a violation of the rules (if this is not apparent, try hypotheticals for each answer; the violation will become apparent when you attempt to place all the variables). Answer choice (C) is incorrect because K and O must be scheduled for consecutive days. Thus, answer choice (D) is proven correct by process of elimination. Overall, this is a very difficult List question—much harder than average.

Get the most out of your LSAT Prep Plus subscription.

Analyze and track your performance with our Testing and Analytics Package.